Re: Un prodotto infinito

Messaggioda gugo82 » 26/07/2016, 13:08

Per il limite hai ragione, dato che mi sono dimenticato un fattore $(a-1)^2$ al denominatore dell'esponente.

Per il resto, facile che abbia sbagliato qualche somma... Capita, facendo i conti a volo. :wink:
Si ha:
\[
\begin{split}
\sum_{n=1}^N \frac{n}{a^n} &= \sum_{n=1}^N \sum_{k=1}^n \frac{1}{a^n} \\
&= \sum_{k=1}^N \sum_{n=k}^N \frac{1}{a^n}\\
&= \sum_{k=1}^N \frac{1}{a^k}\ \sum_{n=0}^{N-k} \frac{1}{a^n}\\
&= \sum_{k=1}^N \frac{1}{a^k}\ \frac{1- \frac{1}{a^{N-k+1}}}{1-\frac{1}{a}}\\
&= \frac{a}{a-1}\ \sum_{k=1}^N \frac{1}{a^k}\ \left( 1 - \frac{a^k}{a^{N+1}}\right)\\
&= \frac{a}{a-1}\ \sum_{k=1}^N \frac{1}{a^k} - \frac{1}{a^{N+1}}\\
&= \frac{a}{a-1}\ \left( \frac{1-\frac{1}{a^{N+1}}}{1-\frac{1}{a}} - 1 - \frac{N}{a^{N+1}} \right)\\
&= \frac{a}{a-1}\ \left( \frac{a^{N+2} - a }{a^{N+1} (a-1)} - 1 - \frac{N}{a^{N+1}}\right)\\
&= \frac{a}{a-1}\ \frac{a^{N+2} - a - a^{N+1} (a-1) - N(a-1)}{a^{N+1} (a-1)}\\
&= \frac{a^{N+1} +(1- a)N -a}{a^N(a-1)^2}
\end{split}
\]
dunque:
\[
\prod_{n=1}^N a^{n/a^n} = a^{\sum_{n=1}^N n/a^n} = a^\frac{a^{N+1} - a(N+1) +N}{a^N(a-1)^2}\; ,
\]
da cui si trae:
\[
\prod_{n=1}^\infty a^{n/a^n} = a^\frac{a}{(a-1)^2}\; .
\]
Sono sempre stato, e mi ritengo ancora un dilettante. Cioè una persona che si diletta, che cerca sempre di provare piacere e di regalare il piacere agli altri, che scopre ogni volta quello che fa come se fosse la prima volta. (Freak Antoni)
Avatar utente
gugo82
Cannot live without
Cannot live without
 
Messaggio: 17392 di 44961
Iscritto il: 12/10/2007, 23:58
Località: Napoli

Re: Un prodotto infinito

Messaggioda Delirium » 26/07/2016, 21:16

Ho un rilancio: mostrare che \[\prod_{n=1}^\infty \frac{(n^2 +1)}{\sqrt{n^4 + 4}} = \sqrt{2}.\]
Ultima modifica di Delirium il 27/07/2016, 11:58, modificato 1 volta in totale.
Delirium
 

Re: Un prodotto infinito

Messaggioda anto_zoolander » 26/07/2016, 21:24

@Delirium

Testo nascosto, perché contrassegnato dall'autore come fuori tema. Fai click in quest'area per vederlo.
Non puoi farmi questo mentre non sono a casa :cry:
Error 404
Avatar utente
anto_zoolander
Moderatore
Moderatore
 
Messaggio: 776 di 9002
Iscritto il: 06/10/2014, 15:07
Località: Palermo

Re: Un prodotto infinito

Messaggioda consec » 27/07/2016, 03:35

Testo nascosto, fai click qui per vederlo
$\prod_{n=1}^\infty \frac{(n^2 +1)}{\sqrt{n^4 + 4}} = \sqrt{2} rArr \lim_{N \to \infty}\prod_{n=1}^\N \frac{(n^2 +1)^2}{n^4 + 4} = 2$

$n_i^4+4=n_i^4+4+4n_i^2-4n_i^2=(n_i^2+2)^2-4n_i^2=(n_i^2+2+2n_i)(n_i^2+2-2n_i)$

Quindi l'i-esimo termine del prodotto può essere scritto nella forma $a^2/(bc)$ dove
$a=n_i^2+1$
$b=n_i^2+2+2n_i$
$c=n_i^2+2-2n_i$

Ma
$(n-1)^2+1=n^2+1-2n+1=n^2-2n+2=c$
$(n+1)^2+1=n^2+1+2n+1=n^2+2n+2=b$

Quindi il $b_i$esimo termine si semplifica con l'$a_(i+1)$esimo numeratore della frazione successiva e il $c_i$esimo termine si semplifica con l'$a_(i-1)$esimo numeratore della frazione precedente. Quindi ogni termine della produttoria tranne il primo e l'ultimo sono uguali a $1$, in quanto il numeratore semplifica il termine $c_i$ del successivo e il termine $b_i$ del precedente, e il denominatore riduce i termini $a_i$ delle frazioni adiacenti. Gli unici fattori a sopravvivere sono l'$a_1$ e il $c_1$ del primo termine, in quanto non c'è nessuna frazione prima, e l'$a_N$ e il $b_N$ dell'ultimo termine, in quanto non c'è nessuna frazione dopo. Di conseguenza.

$\prod_{n=1}^\N \frac{(n^2 +1)^2}{n^4 + 4}=((1^2+1)/(1^2+2*1-2))*((N^2+1)/(N^2+2+2N))=(2(N^2+1))/(N^2+2N+2)$

Passando al limite il risultato è $2$.
consec
Junior Member
Junior Member
 
Messaggio: 16 di 178
Iscritto il: 04/06/2016, 08:47

Re: Un prodotto infinito

Messaggioda anto_zoolander » 27/07/2016, 04:17

@consec

essere anticipato alle 4 di notte :lol: :lol: :lol:

Testo nascosto, fai click qui per vederlo
$S_k=prod_(n=1)^(k)(n^2+1)/sqrt(n^4+4)$


comincio notando che $n^2+4=n^4+4n^2+4-4n^2=(n^2+2n+2)(n^2-2n+2)$ dunque razionalizzo e divido le produttorie in questo modo:

$prod_(n=1)^(k)(n^2+1)/(n^2+2n+2)*prod_(n=1)^(k)sqrt((n^2+2n+2)/(n^2-2n+2))$


ora l'espansione della prima produttoria la possiamo vedere così:

$prod_(n=1)^(k)(n^2+1)/(n^2+2n+2)=2/5*5/10*10/17*...*(k^2-2k+2)/(k^2+1)*(k^2+1)/(k^2+2k+2)$

è chiaro che si semplificano tutti i termini tranne il primo numeratore e il denominatore del $k-esimo$ termine. Ovvero il primo numeratore non è semplificato da nulla, e l'ultimo denominatore non semplifica nulla.

$prod_(n=1)^(k)(n^2+1)/(n^2+2n+2)=2/(k^2+2k+2)$


l'espansione della seconda produttoria invece in questa maniera:

$prod_(n=1)^(k)sqrt((n^2+2n+2)/(n^2-2n+2))=sqrt(5/1)*sqrt(10/2)*sqrt(17/5)*sqrt(26/10)*...*sqrt((k^2-2k+2)/(k^2-6k+10))*sqrt((k^2+1)/(k^2-4k+5))*sqrt((k^2+k+2)/(k^2-2k+2))$

anche quì si nota una regolarità nella semplificazione dei vari termini, dove il numeratore di una frazione si semplifica con il denominatore posto due termini dopo. I primi due denominatori non sono semplificati da nulla e segue che gli ultimi due numeratori non semplificano nulla. Dunque quì il risultato sarà:

$prod_(n=1)^(k)sqrt((n^2+2n+2)/(n^2-2n+2))=sqrt(((k^2+1)(k^2+2k+2))/2)$

dunque infine

$prod_(n=1)^(k)(n^2+1)/sqrt(n^4+4)=2/(k^2+2k+2)*sqrt(((k^2+1)(k^2+2k+2))/2)=sqrt((2k^2+2)/(k^2+2k+2))$


Posto adesso $S_k=sqrt((2k^2+2)/(k^2+2k+2))$ calcolo $lim_(k->+infty)sqrt((2k^2+2)/(k^2+2k+2))$

$lim_(k->+infty)sqrt((2+2/k^2)/(1+2/k+2/k^2))=sqrt2$
Error 404
Avatar utente
anto_zoolander
Moderatore
Moderatore
 
Messaggio: 777 di 9002
Iscritto il: 06/10/2014, 15:07
Località: Palermo

Re: Un prodotto infinito

Messaggioda Delirium » 27/07/2016, 11:55

Non ho controllato i conti, ma mi pare che vada. In fondo si trattava semplicemente di raccapezzarsi delle cancellazioni.

Altri rilanci.

1. Calcolare il valore di \[\prod_{k=2}^\infty \frac{k^3 -1}{k^3 +1}.\]

2. Siano \(a_0 = 5/2\) e \(a_k = a_{k-1} ^2 -2\) per \(k \ge 1\). Calcolare \[\prod_{k=0}^\infty \left(1 - \frac{1}{a_k}\right).\]
Delirium
 

Re: Un prodotto infinito

Messaggioda anto_zoolander » 27/07/2016, 13:30

La prima è semplice

Testo nascosto, fai click qui per vederlo
$prod_(n=2)^(k)(n^3-1)/(n^3+1)=[prod_(n=2)^(k)(n-1)/(n+1)][prod_(n=2)^(k)(n^2+n+1)/(n^2-n+1)]$


Per la prima si ha che l'$(n-2)-esimo$ denominatore si semplifica con l'$n-esimo$ numeratore. Dunque rimarrano solo il primo numeratore e gli ultimi due denominatori.

$prod_(n=2)^(k)(n-1)/(n+1)=2/(k^2+k)$


Per la seconda si ha che l'$(n-1)-esimo$ numeratore si semplifica con l'$n-esimo$ denominatore e quindi rimangono solo il primo denominatore e l'ultimo numeratore.

$prod_(n=2)^(k)(n^2+n+1)/(n^2-n+1)=(k^2+k+1)/3$


$prod_(n=2)^(k)(n^3-1)/(n^3+1)=2/3[(k^2+k+1)/(k^2+k)]$



Ora basta calcolare mandare a limite la produttoria..

$lim_(k->+infty)2/3[(k^2+k+1)/(k^2+k)]=2/3$


Ragiono due minuti sulla seconda e edito.
Ultima modifica di anto_zoolander il 27/07/2016, 14:17, modificato 1 volta in totale.
Error 404
Avatar utente
anto_zoolander
Moderatore
Moderatore
 
Messaggio: 778 di 9002
Iscritto il: 06/10/2014, 15:07
Località: Palermo

Re: Un prodotto infinito

Messaggioda Vincent46 » 27/07/2016, 13:42

Delirium ha scritto:1. Calcolare il valore di \[\prod_{k=2}^\infty \frac{k^3 -1}{k^3 +1}.\]


Testo nascosto, fai click qui per vederlo
Si passa ai logaritmi.
\[\prod_{k=2}^\infty \frac{k^3 -1}{k^3 +1}. = \prod_{k=2}^\infty \frac{(k-1)(k^2+k+1)}{(k+1)(k^2-k+1)}\,. \]

Si ottengono le serie telescopiche

\[ \sum_{k=2}^{n} ( \log (k-1) - \log(k+1) ) = \log(2)-\log(n)-\log(n+1) = \log(2)-\log(n^2+n) \, \]

e, usando il fatto che $k^2+k+1=(k+1)^2-(k+1)+1$,

\[ \sum_{k=2}^{n} ( \log(k^2+k+1)-\log(k^2-k+1) ) = -\log(3)+\log(n^2+n+1) \,.\]

Dunque

\[ \sum_{k^2}^{\infty} \log \frac{(k-1)(k^2+k+1)}{(k+1)(k^2-k+1)} = \log(2)- \log(3) = \log(\frac{2}{3}) \]

e la serie originaria tende a $\frac{2}{3}$.
Vincent46
Average Member
Average Member
 
Messaggio: 93 di 523
Iscritto il: 26/01/2014, 17:27

Re: Un prodotto infinito

Messaggioda Vincent46 » 27/07/2016, 15:10

Delirium ha scritto:2. Siano \(a_0 = 5/2\) e \(a_k = a_{k-1} ^2 -2\) per \(k \ge 1\). Calcolare \[\prod_{k=0}^\infty \left(1 - \frac{1}{a_k}\right).\]


Testo nascosto, fai click qui per vederlo
Per induzione, dimostro che $a_k=2^{2^{k}}+\frac{1}{2^{2^{k}}}$:

$1)$ $a_0 = 2 + \frac{1}{2}$ ;
$2)$ $a_{k+1)=a_k^2-2 = (2^{2^{k}}+\frac{1}{2^{2^{k}}})^2-2=2^{2^{k+1}}+\frac{1}{2^{2^{k+1}}}$.

Dunque si tratta di calcolare

\[ \prod_{k=0}^\infty \left(1 - \frac{1}{2^{2^{k}}+\frac{1}{2^{2^{k}}}}\right). \]

Questo equivale a

\[ \prod_{k=0}^{\infty} \frac{ 2^{2^{k+1}}-2^{2^{k}}+1 }{2^{2^{k+1}}+1 } \]

... ora non so più come andare avanti. :-D forse si può dimostrare la convergenza col il criterio di condensazione di cauchy? in ogni caso non riesco a trovare il valore del prodotto.
Vincent46
Average Member
Average Member
 
Messaggio: 94 di 523
Iscritto il: 26/01/2014, 17:27

Re: Un prodotto infinito

Messaggioda Delirium » 27/07/2016, 15:54

Delirium ha scritto:[...]

2. Siano \(a_0 = 5/2\) e \(a_k = a_{k-1} ^2 -2\) per \(k \ge 1\). Calcolare \[\prod_{k=0}^\infty \left(1 - \frac{1}{a_k}\right).\]

Questa è più tosta. Posto i miei ragionamenti:

Dopo aver fatto qualche conto con i primi valori salta fuori che \(a_k = 2^{2^k} + 2^{-2^k}\) per \(k \in \mathbb{N}\). Inoltre la condizione assegnata porge \(a_k + 1 = a_{k-1} ^2 -1 = (a_{k-1} -1)(a_{k-1} +1)\) donde la produttoria si tramuta in \[\prod_{k=0}^\infty \frac{a_k - 1}{a_k} = \prod_{k=0}^\infty \frac{a_{k+1} + 1}{a_k +1} \cdot \frac{1}{a_k} = \frac{a_1 + 1}{a_0+1} \cdot \frac{1}{a_0} \cdot \frac{a_2 + 1}{a_1 + 1} \cdot \frac{1}{a_1} \cdot \dots \] e sembrerebbe quindi che le produttorie parziali siano \[ \frac{a_{N+1} +1 }{a_0 + 1} \prod_{k=0}^N \frac{1}{a_k}. \]Ora, non mi è chiaro come vada quel prodotto; quello che si riesce a fare è (provare a) stimarlo dall'alto: \[\log \left[ \prod_{k=0}^N \frac{1}{a_k} \right] = \log \left[ \prod_{k=0}^N \frac{2^{2^k}}{2^{2^{k+1}} +1} \right] \le \sum_{k=0}^N \log \left(\frac{1}{2} \right)^{2^k} = \log\left(\frac{1}{2}\right)^{2^{N+1} -1} \]ove ho usato le proprietà del logaritmo e la formula della somma geometrica con ragione \(=2\); quindi \[\prod_{k=0}^N \frac{1}{a_k} \approx \left( \frac{1}{2} \right)^{2^{N+1} -1}. \] Infine una stima è fornita da \[\lim_{n \to \infty} \frac{2^{2^{n+1}} + 2^{-2^{n+1}}+1}{a_0 + 1} \cdot \frac{2}{2^{2^{n+1}}}= 2/(a_0 + 1)=4/7. \]

Non sono riuscito a fare di meglio.

Edit. In effetti, la stima di cui sopra non è così rozza: infatti il valore esatto di convergenza è \(3/7\) (cfr. qui, problema A3). Il problema è preso dall'edizione 2014 della Putnam Mathematical Competition.
Delirium
 

Precedente

Torna a Pensare un po' di più

Chi c’è in linea

Visitano il forum: Nessuno e 1 ospite